Q6

 
alinanny
Thanks Received: 2
Forum Guests
 
Posts: 26
Joined: May 07th, 2011
 
This post thanked 1 time.
 
 

Q6

by alinanny Sat Nov 24, 2012 3:27 pm

I had a hard time eliminating E. I think I am making too big of a logic leap to assume that the fact that books will be stored digitally will be the primary reason why the new digital model will be accepted. Maybe is the lower cost or how easy it will be to print on the spot but not because we will be more familiar with the technology that digitized all books. Even that is an assumption I am making.
Any clearer explanation? Thank you
 
giladedelman
Thanks Received: 833
LSAT Geek
 
Posts: 619
Joined: April 04th, 2010
 
This post thanked 2 times.
 
 

Re: Q6

by giladedelman Sun Nov 25, 2012 8:07 pm

Note: this post has been updated after the original poster pointed out that I made a mistake!

You're right that (E) requires an unwarranted assumption; put another way, the passage does not support the idea that there is "widespread familiarity with new ways of storing information." It's just not in the passage.

(C) is correct because the last paragraph tells us that "the competitive pressure" spurred by newer firms that can respond to literary agents' demands will force traditional publishers to shift over to digital publishing.

Or, as you said in another post, "traditional publishers will be hesitant to change to the new model but will eventually do so because they could lose authors if they did not change. Literary agents could demand more $ for the authors (which those subscribed to the new method will be able to pay) forcing the others to agree to the new model." Well put!

(A) is out because "changing literary tastes" is totally out of scope.

(B) is incorrect because although a perk of the new technology is to make it easier to keep books in print, this is not the reason why publishers will adopt that technology.

(D) is incorrect because the passage never talks about innovative marketing strategies.

Hope that helps!
 
asafezrati
Thanks Received: 6
Atticus Finch
Atticus Finch
 
Posts: 116
Joined: December 07th, 2014
 
 
 

Re: Q6

by asafezrati Sat Aug 08, 2015 5:41 pm

The language in the wrong answer choices is also very strong and demand that the passage will mirror that language (which it doesn't).

A. "the main cause"
B. "primary factor"
D. "ensure"
E. "primary reason"

But I'm not sure about C - "impetus"
It looks strong, but not like the others.
User avatar
 
tommywallach
Thanks Received: 468
Atticus Finch
Atticus Finch
 
Posts: 1041
Joined: August 11th, 2009
 
 
 

Re: Q6

by tommywallach Fri Aug 14, 2015 3:31 pm

I would argue "impetus" is about the same as the others, if you look at the dictionary definition.
Tommy Wallach
Manhattan LSAT Instructor
twallach@manhattanprep.com
Image
 
DominicE471
Thanks Received: 0
Vinny Gambini
Vinny Gambini
 
Posts: 8
Joined: June 08th, 2024
 
 
 

Re: Q6

by DominicE471 Fri Aug 09, 2024 1:49 pm

tommywallach Wrote:I would argue "impetus" is about the same as the others, if you look at the dictionary definition.


Facts.